Reverse Product Rule: Understanding its Application in Integration

Click For Summary
The discussion focuses on the application of the reverse product rule in integration, specifically using the formula ∫uv' = uv - ∫vu'. A participant expresses confusion regarding their teacher's notation, where a and b are assigned as u and v, respectively, and questions whether this was intentional. Another participant clarifies that the correct integration by parts formula is ∫udv = uv - ∫vdu, emphasizing the importance of including the variable of integration. They suggest that if integrating a*b, one should set u = a and dv = bdx, then differentiate and integrate accordingly. Understanding the proper application of these rules is crucial for successful integration.
brandy
Messages
156
Reaction score
0

Homework Statement


i'm sorry i know I've been bombarding physics forums with questions but i need help :p
using reverse product rule \int uv' = uv - \int vu'
and say i have a*b
i noticed my teaher said that a=u and b=v not v' and he simly made that into a v' by deriving.
is there a point to this if so what is the point and is this something he is likely to have done on purpose or by accident.


Homework Equations


...and yes a*b = e^-y /y

The Attempt at a Solution


i attempted, but failed miserably.
 
Physics news on Phys.org
One of the things you are NOT doing that you should is including the "dx" or "dt" or whatever the variable of integration is. The correct statement of "integration by parts", which is, as you say, the "reverse product rule", is \int udv= uv- \int vdu.

Now, I don't know what you mean by "say I have a*b". Do you mean you are trying to integrate \int a(x)b(x)dx? In that case, you could try u= a, dv= bdx, then find du by differentiating and find v by integrating.
 
Question: A clock's minute hand has length 4 and its hour hand has length 3. What is the distance between the tips at the moment when it is increasing most rapidly?(Putnam Exam Question) Answer: Making assumption that both the hands moves at constant angular velocities, the answer is ## \sqrt{7} .## But don't you think this assumption is somewhat doubtful and wrong?

Similar threads

  • · Replies 9 ·
Replies
9
Views
2K
  • · Replies 4 ·
Replies
4
Views
2K
  • · Replies 6 ·
Replies
6
Views
2K
  • · Replies 3 ·
Replies
3
Views
2K
  • · Replies 4 ·
Replies
4
Views
2K
  • · Replies 9 ·
Replies
9
Views
3K
  • · Replies 1 ·
Replies
1
Views
1K
  • · Replies 4 ·
Replies
4
Views
3K
Replies
6
Views
2K
  • · Replies 1 ·
Replies
1
Views
2K